(-8)+8-(-7)

answer? please help

Answers

Answer 1
-8 and +8 equal 0 since they cancel out. It's basically 8-8. So that leaves you with 0-(-7). The two negatives make a positive, now you're left with 0+7=7. Your answer is 7. Hope this helps :)
Answer 2

Answer:

[tex]\Huge \boxed{\mathrm{7}}[/tex]

Step-by-step explanation:

[tex](-8)+8-(-7)[/tex]

Distributing the negative sign to the term in the bracket.

[tex]\Rightarrow (-8)+8+7[/tex]

Combining.

[tex]\Rightarrow 7[/tex]


Related Questions

sin(7pi/6)=
a. 1/2
b. -sq3/2
c.sq3/2
d.-1/2

Answers

Answer:

it IS b

Step-by-step explanation:

A. Use composition to prove whether or not the functions are inverses of each other. B. Express the domain of the compositions using interval notation.

Answers

Answer:

Step-by-step explanation:

A). If a function 'f' is inverse of another function 'g',

Then f[g(x)] = x = g[f(x)]

In this question the given functions are,

f(x) = [tex]\frac{1}{x-3}[/tex] and g(x) = [tex]\frac{3x+1}{x}[/tex]

Then, f[g(x)] = [tex]\frac{1}{\frac{3x+1}{x}-3}[/tex]

                    = [tex]\frac{x}{3x+1-3x}[/tex]

                    = x

Similarly, g[f(x)] = [tex]\frac{(\frac{3}{x-3})+1}{\frac{1}{x-3} }[/tex]

                         = [tex]\frac{3+x-3}{1}[/tex]

                         = x

Therefore, Both the functions are inverse of each other.

B). Domain of the compositions of these functions will be a set of all real numbers, (-∞, ∞)

I need help someone please help me

Answers

A

Step-by-step explanation:

Answer:

the answer is 14. hope this helps :)

Step-by-step explanation:

PLEASE HELP! I will mark as brainliest if you help me. Solve for x. −ax + 2b > 8

Answers

Answer:

x < ( 8 - 2b ) / -a

Step-by-step explanation:

For this problem, we simply will solve the inequality for x.

-ax + 2b > 8

-ax > 8 - 2b

x < ( 8 - 2b ) / -a

Note, the comparison is flipped since a negative multiplicand was used to isolate x.

Cheers.

An object is moving at a speed of 3 kilometers per hour. Express this speed in feet per
minute. Round your answer to the nearest whole number.

Answers

The speed of the object in feet per minute to nearest whole number is 164feet/min

Speed is the ratio of the distance to the time of an object. Mathematically;

Speed = Distance/Time

Given the following

Speed = 3km/hr

Using the conversion ratio

1km = 3280.84feet

3km = x

Cross multiply

x = 3 * 3280.84

x = 9,842.52feet

Convert 1 hour to minute

1 hour = 60minutes

Get the speed in feet per minutes

Speed in feet/minutes = [tex]\frac{9,842.52}{60}[/tex]

Speed in feet/minutes = 164.042 feet/minutes

Hence the speed of the object in feet per minute to nearest whole number  is 164feet/min

Learn more here: https://brainly.com/question/1462592

The speed in feet per  minute is 164.

Given the following data:

Speed = 3 kilometers per hour.

In this exercise, you're required to convert the value of the speed given in kilometers per hour to feet per minute.

Conversion:

1 kilometer = 3280.84 feet

3 kilometer = X feet

Cross-multiplying, we have;

X [tex]= 3280.84[/tex] × [tex]3[/tex]

X = 9842.50 feet

Since we know that one (1) hour is equal to sixty (60) minutes, we would divide the value of X by 60 in order to convert it to feet per minute.

[tex]X = \frac{9842.50}{60}[/tex]

X = 164.042 feet per minute.

To the nearest whole number;

X = 164 feet per minute

Therefore, the speed in feet per  minute is 164.

Find more information: https://brainly.com/question/24457218

What fraction of an hour is 32 minutes? please let me know! :(

Answers

Answer:

8/15

Step-by-step explanation:

1 hour = 60 minutes

32 minutes / 60 minutes

Divide top and bottom by 4

8/15

To estimate the percentage of defects in a recent manufacturing batch, a quality control manager at Toshiba selects every 12th laptop that comes off the assembly line starting with the ninth until she obtains a sample of 110 laptops. What type of sampling is used?

a. Simple random
b. Stratified
c. Convenience
d. Systematic
e. Cluster

Answers

Answer:

D) systematic

Step-by-step explanation:

Systematic sampling. With systematic random sampling, one creates a list of every member of the population, and from the list, one can randomly select the first sample element from the first Nth elements on the population list. Thereafter, we select every Nth element on the list. Selection is done according to a random starting point but with a fixed, periodic interval. This interval, which is called the sampling interval, is calculated by dividing the population size by the desired sample size.

The segment JL has a definite length of [Drop Down 1]. Segments JK and KL have both definite lengths of [Drop Down 2] and [Drop Down 3], respectively. Both segments are [Drop Down 4]. JL=5x-9, JK=3x and KL=1x+4

Answers

Answer:

[tex]JL=56[/tex]       [tex]JK=39[/tex]        [tex]KL=17[/tex]

Step-by-step explanation:

Your question is not well presented/formatted; however, I can deduce that the question requires that you calculate the following lengths;

JK, KL and JL

Given that

[tex]JL=5x-9[/tex]

[tex]JK=3x[/tex]

[tex]KL=1x+4[/tex]

The relationship between the given parameters is:

[tex]JL = JK + KL[/tex]

Substitute 5x - 9 for JL; 3x for JK and 1x + 4 for KL

[tex]5x - 9 = 3x + 1x + 4[/tex]

Collect Like Terms

[tex]5x - 3x - 1x = 9 + 4[/tex]

[tex]x = 13[/tex]

Now that the value of x has been solved, we can easily get the numeric values of JK, KL and JL by substituting 13 for x

So;

[tex]JL=5x-9[/tex]

[tex]JL=5*13-9[/tex]

[tex]JL=65 - 9[/tex]

[tex]JL=56[/tex]

[tex]JK=3x[/tex]

[tex]JK=3 * 13[/tex]

[tex]JK=39[/tex]

[tex]KL=1x+4[/tex]

[tex]KL=1 * 13+4[/tex]

[tex]KL=13+4[/tex]

[tex]KL=17[/tex]

Hence;

[tex]JL=56[/tex]       [tex]JK=39[/tex]        [tex]KL=17[/tex]

Very confused. I need help with this question:

-7=M/7

Answers

Answer:

M=-49

Step-by-step explanation:

-7=M/7

-7*7=M

-49=M

Answer:

-7= -49/7

Step-by-step explanation:

you're just creating an equal expression so think of it as -7/1 = -49/7. your just multiplying the first ratio by 7 but when you simplify the two ratios theyu are equal

1. Carl has two dogs. They are black Labrador retrievers. The male weighs 31.75 kg and the female weighs 29.48 kg.
a. How much heavier is the male than the female? Show your work.
b. How much do they weigh together? Show your work.​

Answers

Answer:

A. 2.27 kg

B. 61.23 kg

Step-by-step explanation:

A. 31.75 - 29.48 = 2.27

B. 31.75 + 29.48  = 61.23

PART A :A deli receives a large order for sandwiches for a company picnic. The company wants one club sandwich and two regular sandwiches for each person. The company also wants an extra Dagwood sandwich. A club sandwich has $3$ slices of bread. A regular sandwich has $2$ slices. A Dagwood has $5$ slices. If there are $n$ people going to the company picnic, how many slices of bread does the deli need to make this order? Write your answer as a fully simplified expression. Your answer should have the variable $n$ in it exactly once. PART B: Given that the deli ends up using $110$ slices of bread, how many people are going to the company picnic?

Answers

Answer:

Part A:

[tex]7n + 5[/tex]

Part B:

15 people

Step-by-step explanation:

Each person gets one club sandwich and two regular sandwiches. There is one additional Dagwood sandwich.

If a club sandwich has 3 slices, then the total amount of slices needed for [tex]n[/tex] people will be [tex]3n[/tex], since each person has 3 slices.

If a regular sandwich has 2 slices and everybody gets 2 regular sandwiches, then it's represented as [tex]4n[/tex], since everyone has 4 slices each.

Additionally, we have one Dagwood sandwich, which can be represented as 5,, since a Dagwood sandwich has 5 slices.

Combining these terms we get [tex]3n + 4n + 5[/tex]. Combining like terms gets us the expression [tex]7n + 5[/tex].

To solve for  [tex]7n + 5[/tex], we have to make it an equation. Using the information in Part B, the Deli ends up receiving 110 slices of bread. So, the equation is

[tex]7n + 5=110[/tex]

We want to isolate n on one side, to do this we have to subtract 5 then divide by 7.

[tex]7n + 5 - 5= 110-5\\\\7n = 105\\\\7n\div7 = 105\div7\\\\n = 15[/tex]

So 15 people are going to the picnic.

Hope this helped!

Find the deriative dy/dx for y=x^2-2x/x^3+3

Answers

Answer:

[tex]\frac{dy}{dx}=(\frac{(2x-2)(x^3+3)-(x^2-2x)(3x^2)}{(x^3+3)^2})[/tex]

Step-by-step explanation:

So we want to find the derivative of the rational equation:

[tex]y=\frac{x^2-2x}{x^3+3}[/tex]

First, recall the quotient rule:

[tex](\frac{f}{g})'=\frac{f'g-fg'}{g^2}[/tex]

Let f be x^2-2x and let g be x^3+3.

Calculate the derivatives of each:

[tex]f=x^2-2x\\f'=2x-2[/tex]

[tex]g=x^3+3\\g=3x^2[/tex]

So:

[tex]\frac{dy}{dx}=(\frac{x^2-2x}{x^3+3})'[/tex]

Use the above format:

[tex]\frac{dy}{dx}=\frac{f'g-fg'}{g^2}\\\frac{dy}{dx}=(\frac{(2x-2)(x^3+3)-(x^2-2x)(3x^2)}{(x^3+3)^2})[/tex]

And that's our answer :)

(If you want to, you can also expand. However, no terms will be canceled.)

Find the missing lengths of the sides

Answers

Y= 8 square root of 2
X= 8
The answer should be Y=8,x=8

The product of five and a number, increased by 9 is equal to fifteen less than three times then
number. Write an equation, solve, and find the number.

Answers

Answer:

x = -12

Step-by-step explanation:

Let x be the number.

The product of 5 and x increased by 9 is equal to 15 less than 3 times x.

5x + 9 = 3x - 15.

We can isolate x, getting (5-3)x = -15-9

Then we can solve:

2x = -24

x = -12.

The required number is -12.

What is an equation?

An equation is a mathematical statement that shows that two mathematical expressions are equal.

Given that, The product of five and a number, increased by 9 is equal to fifteen less than three times then number.

Let x be the number.

Establishing the equation,

5x + 9 = 3x - 15.

(5-3)x = -15-9

Then we can solve:

2x = -24

x = -12.

Hence, the number is -12

For more references on equations, click;

https://brainly.com/question/29657988

#SPJ2

-8(6-r)=24 what is the solution for r

Answers

Answer:hi

Step-by-step explanation:Let me first warn you that I CANNOT get the denominators of this problem to go underneath the numerators. It looks good on my answer page but when I click "preview your answer" the denominators are ALL over the place. Grrrrr.

I think the weirdly placed denominators make this explanation hard to understand but you give this a go and see if you understand things. If it is too hard to see, maybe this website will let you instant message me or email me. If so, then I will send you the solution via email. I'm very sorry the denominators are SO scattered.

See what you think of the explanations below but it might help if you write down my answer on paper, so that you can move the number to rest more closely under their proper numerators. :-)

Ok then......

Let's start with your first problem. A quick way to do this work is to cross multiply. Here's what I mean:

__3___ = __6___

r - 1 r + 2

Ok, so you can cross multiply:

3(r + 2) = 6(r - 1) (cross multiply)

3r + 6 = 6r - 6 (use the distributive property)

Now you will have to put the "r" variables on the left side of the equal sign and numbers without variables on the right side of the equal sign. To get 6r from the right side to the left, you can subtract 6r from the right side. If you subtract it from the right, you must subtract it from the left.....

To get 6 from the left side to the right, you can subtract it from the left. If you subtract it from the left, you must subtract it on the right.....

So now you have:

3r - 6r = -6 - 6 This becomes.........

- 3r = -12

Now to solve for "r" you will have to divide by -3. Why? Well, -3r is really saying: -3 times r. The -3 and the r are actually being multiplied together so to isolate the "r" you have to sort of "undo" the multiplication. What is the opposite of multiplication? It's DIVISION. Soooo, you must divide by -3 on both sides. SO you have:

- 3r = -12

r = 4 (-12 divided by -3 is a positive 4)

Now you know "r" equals 4. If we "plug" 4 back into the original equation, we can see the answer is correct. Check this out:

__3___ = __6___ (original equation)

r - 1 r + 2

__3___ = __6___ (substitute "4" in for "r"

4 - 1 4 + 2

__3___ = __6___ (do the math in the denominator area)

 

3 6

1 = 1 (3/3 equals 1 and 6/6 also equals 1, so our

solution that r = 4 is correct!) :-)

Now to your 2nd problem..........

4/2x-5 = 24/2x

___4___ = __24___

2x � 5 2x

Let's cross multiply again:

4(2x) = 24(2x - 5)

8x = 48x - 120

8x - 48x = - 120

-40x = - 120

(divide both sides by -40)

x = 3

Now let's "plug" 3 in where we had the variable "x" in the original equation to see if we are correct.

___4___ = __24___ (original equation)

2x � 5 2x

___4___ = __24___ (substituted 3 for "x")

2(3) � 5 2(3)

___4___ = __24___

6 - 5 6

___4___ = __24___

1 6

4 = 4

SO once again we have found the correct answer. Yay for us! :-)

I hope this helps. :-)

Answer:

[tex]r=9[/tex]

Step-by-step explanation:

So we have the equation:

[tex]-8(6-r)=24[/tex]

Divide both sides by -8:

[tex]6-r=-3[/tex]

Subtract 6 from both sides:

[tex]-r=-9[/tex]

Divide both sides by -1:

[tex]r=9[/tex]

And we're done!

Marlon is bowling in a tournament and has the highest average after 5 games, with scores of 210, 225, 254, 231, and 280. In order to maintain this exact average, what must be Marlon's score for his 6th game?

Answers

Answer:

By scoring 240 in the sixth game, Marlon can maintain this exact average

Step-by-step explanation:

The first thing to do here is to calculate the average for the first 5 games.

Mathematically, the average can be calculated by adding all the scores in the games and dividing by 5;

= (210 + 225 + 254 + 231 + 280)/5 = 1200/5 = 240

Now to maintain this average score of 240, we want to know the score she must have in the 6th game.

Let the score be x;

By adding x to the previous total of 1200 before and dividing by the new count of 6, we can work our way to get x.

Mathematically, what we are saying is;

(1200 + x)/6 = 240

6 * 240 = 1200 + x

1440 = 1200 + x

x = 1440 -1200

x = 240

What is the solution of the equation below? y/5- = -7

Answers

Answer:

y=-35

Step-by-step explanation:

Steps

$\frac{y}{5}-=-7$

$\mathrm{Subtract\:}-\mathrm{\:from\:both\:sides}$

$\frac{y}{5}=-7$

$\mathrm{Simplify}$

$\frac{y}{5}=-7$

$\mathrm{Multiply\:both\:sides\:by\:}5$

$\frac{5y}{5}=5\left(-7\right)$

$\mathrm{Simplify}$

$y=-35$

Answer:

y = 35

Step-by-step explanation:

    multiply by -5 on both sides

    -5 and -5 cancel out

    -5 times -7 is 35

    y = 35

Wakaba buys some granola bars at $0.50 each and energy drinks at $2 each for a group hike. She buys twice as many granola bars as energy drinks. If she spends $27 in total, how many granola bars and energy drinks does she buy?

Answers

Answer: 9 energy drinks and 18 granola bars.

Step-by-step explanation:

Price per granola bar = $0.50

Price per energy drink = $2.

We can define G as the number of granola bars that she bought, and D as the number of drinks that she bought.

Then

"She buys twice as many granola bars as energy drinks."

G = 2*D

" If she spends $27 in total"

G*$0.50 + D*$2 = $27

So we have a system of equations:

G = 2*D

G*$0.50 + D*$2 = $27

The first step will be to replace the first equation into the second one, and solve it for D.

(2D)*$0.50 + D*$2 = $27

D*(2*$0.50) + D*$2 = $27

D*$1 + D*$2 = $27

D*$3 = $27

D = $27/$3 = 9.

So she bought 9 energy drinks, and we know that:

G = 2*D

G = 2*9 = 18

She bought 18 granola bars.

Any one smart enough to answer me (a-b) (a+2b) ?​

Answers

Answer:

ok lets do it..

Step-by-step explanation:

You simply multiply out each factor.

So you start by multiplying the a in (a+b) to each term in (a−2b).

You will get a2−2ab as the first portion of the answer.

So now you will do the same with the b in (a+b).

Then you get −2b2+ab.

Now you add a2−2ab and −2b2+ab to get a2−2b2−ab .

i hope it will work....

Answer:

a² + ab - 2b²

Step-by-step explanation:

Given

(a - b)(a + 2b)

Each term in the second factor is multiplied by each term in the first factor, that is

a(a + 2b) - b(a + 2b) ← distribute both parenthesis

= a² + 2ab - ab - 2b² ← collect like terms

= a² + ab - 2b²

facorts of 8 pleaseeeeeeeeeeeeeeeeeeeeee

Answers

Answer :1,2,4,8

Step-by-step explanation:

Answer:

Factors  Multiples

1, 2, 3, 6 6 60

1, 7 7 70

1, 2, 4, 8 8 80

1, 3, 9 9 90

Step-by-step explanation:

factors of 8 are numbers between -8 and 8 because all of these numbers are between 8 and -8 it has to have split between two numbers thats how you get factors of 8.

answer the question quick​

Answers

Answer:

x^2 +6x -15

Step-by-step explanation:

(3x^2 +2x -8) - (2x^2 -4x+7)

Distribute the minus sign

(3x^2 +2x -8) - 2x^2 +4x-7

Combine like terms

x^2 +6x -15

Answer:

x²+6x-15

Step-by-step explanation:

We are given this expression:

[tex](3x^2+2x-8) - ( 2x^2-4x+7)[/tex]

We want to simplify the expression. First, distribute the negative sign in front of the second set of parentheses. Multiply each term inside the parentheses by -1.

[tex](3x^2+2x-8) + (-1*2x^2) + (-1*-4x) + (-1*7)[/tex]

[tex](3x^2+2x-8) -2x^2+ 4x -7[/tex]

Next, combine like terms. The 2 terms with an x² can be combined, just like the two terms with an x, and the two terms without a variable.

[tex](3x^2-2x^2) + (2x+4x) + (-8-7)[/tex]

[tex]x^2+(2x+4x)+(-8-7)[/tex]

[tex]x^2+6x+(-8-7)[/tex]

[tex]x^2+6x-15[/tex]

The expression x²+6x-15 is equivalent to (3x²+2x-8)-(2x²-4x+7)

SA= 2 pie r square + 2 pie rh, solve for h please ​

Answers

Answer:

[tex]\huge \boxed{h= \displaystyle \frac{SA}{2\pi r} -r}[/tex]

[tex]\rule[225]{225}{2}[/tex]

Step-by-step explanation:

[tex]SA=2\pi r^2 +2\pi rh[/tex]

Subtracting [tex]2\pi r^2[/tex] from both sides:

[tex]SA-2\pi r^2 =2\pi rh[/tex]

Dividing both sides by [tex]2\pi r[/tex]:

[tex]\displaystyle \frac{SA-2\pi r^2}{2\pi r} =h[/tex]

Simplifying the equation:

[tex]\displaystyle \frac{SA}{2\pi r} -r=h[/tex]

[tex]\rule[225]{225}{2}[/tex]

The sum of two numbers is 80. The difference between the number is 14. Find the numbers

Answers

Answer:

80 -14=66 and 66-80=52 if we check the 66-52 the difference will be 14

4. A rock is placed in a graduated cylinder with 40-mL of water in it. The water level rises to 60-mL. When placed on a balance scale it takes 54 grams of gram stackers to balance the scale. What is the density of the rock?

Answers

Answer:

2.7 [tex]g/mL[/tex]

Step-by-step explanation:

The density of an object will be its mass over volume.

We know that the mass of this rock is 54 grams because it took 54 grams of gram stackers to balance the scale. If we have a balanced scale, that means the amount of weight on either side is equal. So the mass is 54 grams.

Also, we can tell that the volume of the rock is 20 mL. This is because, according to Archimedes principle, if you put an object in water, the amount of water displacement will be it's volume. So the amount of water displacement is [tex]60-40=20[/tex] mL.

Density is mass over volume, so the density is [tex]54\div20 = 2.7[/tex]. Density is measure in mass over volume, so [tex]g/mL[/tex].

So 2.7  [tex]g/mL[/tex] is the density of the rock.

Hope this helped!

What is the real number value of m^3+ √12 m when 5m^2 = 45 ?
F. 15
G. 27
H. 33
J. 38.09
K. 739.39

Answers

Answer: H. 33

=============================================

Work Shown:

Solve 5m^2 = 45 for m to get

5m^2 = 45

m^2 = 45/5

m^2 = 9

m = sqrt(9)

m = 3

I'm making m to be positive so that way the expression 12m is not negative. Otherwise, sqrt(12m) would not be a real number result.

--------------

Plug m = 3 into the expression we want to evaluate

m^3 + sqrt(12m)

3^3 + sqrt(12*3)

27 + sqrt(36)

27 + 6

33

3/5 of a number is 162. Work out the number. How do I do this AQA question?

Answers

Answer:

The number is 270

Step-by-step explanation:

Let the number be 'x'

3/5 of x = 162

[tex]\frac{3}{5}*x = 162\\\\x=162*\frac{5}{3}\\\\x=54 * 5\\\\x = 270[/tex]

Answer:

[tex] \boxed{ \bold{ \huge{ \boxed{ \sf{270}}}}}[/tex]

Step-by-step explanation:

Let the number be 'x'

[tex] \sf{ \frac{3}{5 } \: \: of \: x \: = 162}[/tex]

⇒[tex] \sf{ \frac{3x}{5} = 162}[/tex]

Apply cross product property

⇒[tex] \sf{3x = 162 \times 5}[/tex]

Multiply the numbers

⇒[tex] \sf{3x = 810}[/tex]

Divide both sides of the equation by 3

⇒[tex] \sf{ \frac{3x}{3} = \frac{810}{3} }[/tex]

Calculate

⇒[tex] \sf{x = 270}[/tex]

Hope I helped!

Best regards!!

What is the volume of a cylinder that is 15-m tall and has a radius of 3 m. Use 3.14 for π, and round your answer to the nearest cubic meter.

A. 141 m3
B. 283 m3
C. 339 m3
D. 424 m3

Answers

Answer:

D. 424 m³

Step-by-step explanation:

Use the formula for the volume of a cylinder

V = [tex]\pi[/tex]r²h

Plug in the values we know

V = (3.14)(3²)(15)

V = 423.9 m³

This is closest to answer choice D, 424 m³

So, D is correct

The mother bottlenose dolphin weighs
440 pounds. She weighs 250 pounds
more than her calf. How much do they
weigh together?

Answers

Try , 630 pounds because she weighs 250 pounds more than her calf so do 440-250 and you get 190 then add 440 + 190 and you get 630.

Alek is taking an inventory of styles of compression bandages for work. Here is the data he has collected.


The individuals in this data set are:Choose 1 answer:
A) Blood Centers
B) Compression bandage styles
C) Nurses

This data set contains:Choose 1 Answer:
A) 3 variables, 1 of which is categorical
B) 3 variables, 3 of which are categorical
C) 6 variables, 1 of which categorical
D) 6 variables, 3 of which are categorical

Answers

The individuals in this data set are B) Compression bandage styles

The style tags in the first column is what Alex is surveying.

====================================================

This data set contains A) 3 variables, 1 of which is categorical

The three variables are

WidthTotal lengthColor

The first two are quantitative data. We can do math on them such as finding the average. The last variable is categorical. We cannot apply the average as the average color doesn't really make sense. We simply have a list of labels, so this data is nominal. It is not ordinal because we cannot sort the colors.

Answer:

compression bandages styles

3 variables, 1 of which is categorical

15th and 17th one pls
1st Correct answer will be marked as brainiest

Answers

Answer:

Answer of 15(i) is the picture . 15(ii) and 17 are solved

Step-by-step explanation:∆ABC where B=90°

→D is mod point of BC so CD=BD

To Prove:

→BC^2=4(AD^2-AB^2)

PROOF:--

In ∆ABD by pythagorus theorem we get,

AD^2=AB^2+BD^2

SO

➡️BD^2= AD^2-AB^2. (1)

NOW ,

BC= BD+CD

BC= 2BD (AS D IS MID POINT)

SQUAREING BOTH SIDES

BC^2= 4BD^2

BY EQUATION 1 WE GOT BD^2 = AD^2-AB^2

Putting this value

➡️BC^2= 4(AD^2-AB^2)

HENCE PROVED

-------------------------------------------------------------------------

17.

Construction : Draw AE ⊥ BC

Proof : In ∆ABE and ∆ACE, we have

AB = AC [given]

AE = AE [common]

and ∠AEB = ∠AEC [90°]

Therefore, by using RH congruent condition

∆ABE ~ ∆ACE

⇒ BE = CE

In right triangle ABE.

AB2 = AE2 + BE2 ...(i)

[Using Pythagoras theorem]

In right triangle ADE,

AD2 = AE2 + DE2

[Using Pythagoras theorem]

Subtracting (ii) from (i), we get

AB2 - AD2 = (AE2 + BE2) - (AE2 + DE2)

AB2 - AD2 = AE2 + BE2 - AE2 - DE2

⇒ AB2 - AD2 = BE2 - DE2

⇒ AB2 - AD2 (BE + DE) (BE - DE)

But BE = CE [Proved above]

⇒ AB2 - AD2 = (CE + DE) (BE - DE)

= CD.BD

⇒ AB2 - AD2 = BD.CD

Hence Proved.

Other Questions
Right triangle XYZ, with vertex Y at (-3, 0), is rotated clockwise 180 about the origin. Which of the following rotations is equivalent to 180 clockwise rotation about the origin? clockwise 360 counterclockwise 180 counterclockwise 360 clockwise 270 A tapeworm living in a students intestines is a muaalism, commensalism, or parasitism give an example of a Nutritionist might a in order to plan/creates a 25 year old female powerlifter who is wanting to complete in a weight loss program (she wants to move down a weight class to complete) Lloyd Inc. had sales of $200,000, a net income of //415,000, and the following balance sheet: Cash$10,000Accounts Payable$30,000Receivables 50,000Notes Payable To Bank 20,000Inventories 150,000Total Current Liabilities $50,000Total Current Assets$210,000Long-Term Debt 50,000Net Fixed Assets 90,000Common Equity 200,000Total Assets$300,000Total Liabilities And Equity$300,000The new owner thinks that inventories are excessive and can be lowered to the point where the current ratio is equal to the industry average, 2.5x, without affecting sales or net income. If inventories are sold and not replaced (thus reducing the current ratio to 2.5x); if the funds generated are used to reduce common equity (stock can be repurchased at book value); and if no other changes occur, by how much will the ROE change? What will be the firms new quick ratio? Select the correct answer.What are the domain and range of this function?(x + 3) - 5A Domain: (0o,00)Range (-00,00)Domain: (-5,00)Range: (-5,00)C. Domain: (-0,00)Range: (-5,00)D. Domain: (-5,00)Range: (-00,00)ResetNext Early river valley civilizations emerged around which river? Orographic lifting occurs where sloping terrains, such as mountains, act as barriers to the flow of air.a) trueb) false The current standard for RFID is based off of Group of answer choices MIT standards IEEE standards ISO standards there is no agreement on a universal standard thats accepted by all parties Write an expression for the quotient of b divided by the difference of two minus three. Brickhouse is expected to pay a dividend of $3.15 and $2.46 over the next two years, respectively. After that, the company is expected to increase its annual dividend at 3.9 percent. What is the stock price today if the required return is 11.3 percent? what are three core elements of yoga Ivanhoe Inc. issues $240,000, 10-year, 8% bonds at 96. Prepare the journal entry to record the sale of these bonds on March 1, 2020. 8-|21 - 13 - 9| -| -4| It takes Serina 4.44 hours to drive to school. Her route is 15 km long. What is Serinas average speed on her drive to school? To say that the difference between sample averages is statistically To say that the difference between sample averages is statistically significant means that the difference is:A) probably NOT due to chance variation.B) arbitrary.C) of practical importance.D) relatively large. Convert this number expressed in standard form to scientific notation. 28,500,000 1. Move the decimal to create the coefficient: 2.8500000 2. Write in scientific notation: 2.85 10x What is the value of x in the scientific notation? x = How much money did she earn? Dawn is a graduate student who is investigating the development of fine motor skills. She selects one group of children and assesses their fine motor skills every six months over a two-year period. In this example, Dawn is using a _____ research design. help plss I don't speak Spanish it confusing!!!!! Which role does money have in economic systems?aba good to consumea measure of satisfactiona resource for productiona medium of exchanged